An Aquarium’s dimensions are 3 1/4 ft x 2 ft x 1 3/4. What is the volume of the Aquarium?

Answers

Answer 1

Volume = length x width x height

Volume = 3 1/4 x 2 x 1 3/4 = 11 3/8 cubic feet

Answer 2

Answer:

11 3/8 ft^3

Step-by-step explanation:

The volume is given by

V = l*w*h

V = 3 1/4 * 2 * 1 3/4

Change the number to improper fractions

V = (4*3+1)/4  *2 * (4*1+3)/4

   = 13/4 *2* 7/4

Rewriting

   = 13/1 * 2/4 * 7/4

    = 13/1 * 1/2 *7/4

    = 91/8

Changing back to a mixed number

8 goes into 91  11 times with 3 left over

11 3/8 ft^3


Related Questions

Please help solve this. 10points worth it

Answers

Answer:

The function is a growth function so I guess A, B and D

suggest a growth curve

Step-by-step explanation:

Use linear regression to predict the length of a 40-month-old animal. The ages and lengths of several animals of the same species are recorded in the following table: (2 points)

Answers

Answer:

Aw what animal

Step-by-step explanation:

Srry I need pointse

What is the solution to the equation below? (round your answer two decimal places)
e^x=7.1

Answers

Answer:

Step-by-step explanation:

In order to undo that e, you need to take the natural log of both sides:

[tex]ln(e^x)=ln(7.1)[/tex] the ln and the e cancel each other out, leaving us with

x = ln(7.1) so

x = 1.96

Multiply negative 1 over 3 multiplied by 1 over 4. Which of the following is correct? negative 1 over 3 negative 1 over 4 negative 1 over 7 negative 1 over 12

Answers

Answer:

-1/12

Step-by-step explanation:

-1/3 * 1/4

Multiply the numerators

-1 *1 = -1

Multiply the denominators

3 *4 =12

numerator over denominator

-1/12

Answer:

the answer is [tex]\frac{-1}{12} \\[/tex]

Step-by-step explanation:

[tex]\frac{1}{3} x \frac{1}{4}[/tex]

3x4= 12 is now the denominator

                       and

the numerator stays the same because -1x1=-1

so there for the answer is [tex]\frac{-1}{12}[/tex]

hope this helped you  ^.^ happy holiday

I need help It’s honors 7 math pls help

Answers

Answer:

411,600.000× 10^3 OR 4.11600000×10^8

Step-by-step explanation:

The expontent seems to be 3 so you move the decimal point over <this way 3

if it is 8 you move it over this way < 8

Answer:

411,600.000× 10^3 OR 4.11600000×10^8

Step-by-step explanation:

The expontent seems to be 3 so you move the decimal point over <this way 3

if it is 8 you move it over this way < 8

May someone please help??

Answers

Answer:

your inequality signs are correct so i'll skip those

-2.06, .9803

.67, .7486

Step-by-step explanation:

1.)

To do this one you need to standardize x by subtracting the mean and dividng by the standard deviation

(1000-3262)/1100= -2.056 which i will round to -2.06

with this as our z value use the table to find the associated probability

.9803

2.)

Same process

(4000-3262)/1100=.67

which has a probability of .7486

Is the given equation a quadratic equation? Explain. x(x−6)=−5

The equation is not a quadratic equation because there is no x2-term.

The equation is a quadratic equation because there is an x2-term.

The equation is not a quadratic equation because the expression is not equal to zero.

The equation is not a quadratic equation because there is a term with degree higher than 2.

I think the answer is A but im not sure.

Answers

Answer:

The equation is a quadratic equation because there is an x2-term.

Step-by-step explanation:

x(x−6)=−5

Distribute

x^2 -6x = -5

The equation is a quadratic equation because there is an x^2-term.

Answer:

Your required answer is option A.

Step-by-step explanation:

Here,

The given equation is;

x(x-6)=-5

now,

while finding x.

either or,

x=-5 (x-6)=-5

x= 1 (shifting-6 in next side)

now, the value of x is -5,1.

so, it's a quadratic equation.

( in quadratic equation the variable always has two values after solution)

Hope it helps..

can someone explain this please?

Answers

Answer:

3 + 2y = 2 + 4y

Step-by-step explanation:

1. Create the equation

3 + 2y = 2 + 4y

There are 3 "1" blocks and 2 "y" blocks on the left. There are 2 "1" blocks and 4 "y" blocks on the right. The scale shows both are equal.

Answer:

3 + 2y = 2 + 4y

Step-by-step explanation:

To create an equation from this model, we look at both sides of the scale. The left has three 1 units and two y units, and there are two 1 units and four y units on the right side. Since the scale shows that both sides are equal in weight, we know that our equation would have an equal sign to show they are equal.

Left: Three 1 units can be written as 3 and two y units could be written as 2y.

Right: Two 1 units can be written as 2 and four y units could be written as 4y.

Now, we put these values together (with an equal sign) to get:

3 + 2y = 2 + 4y.

Hope this helps!

PLZ Help! I'm crying over this....

Answers

Answer:

D

Step-by-step explanation:

The correct option is the Car stops at a distance away from the starting point because the portion shows a constant function away from the starting point.

From the graph we can tell that the distance of the car is a function and it's a line

Answered by G a u t h m a t h

[tex]-6\left(1-m\right)=9-2m[/tex]

Answers

Answer:

[tex]m=\frac{15}{8}[/tex]

Step-by-step explanation:

[tex]-6\left(1-m\right)=9-2m[/tex]

[tex]-6(1-m)=-6+6m[/tex]

[tex]-6+6m=9-2m[/tex]

Add 6 to both sides:-

[tex]-6+6m+6=9-2m+6[/tex]

[tex]6m=-2m+15[/tex]

Now, add 2m to both sides:-

[tex]6m+2m=-2m+15+2m[/tex]

Divide both sides by 8:-

[tex]\frac{8m}{8}=\frac{15}{8}[/tex]

[tex]m=\frac{15}{8}[/tex]

-----------------------

OAmalOHopeO

-----------------------

Answer:

your answer is m=15/8

The length of a box is 1 cm more than its width. The height of the box is 8 cm greater than the width. The dimensions can be represented by x, x + 1, and x + 8. Multiply the dimensions and find the greatest common factor of the terms.

Answers

Answer:

The greatest common factor is x

Step-by-step explanation:

Dimensions: x,x+1,x+8

Multiplication of thee dimensions:

⇒ Product      =   [tex]x(x+1)(x+8)\\x(x^{2}+8x+1x+8)\\x(x^{2} +9x+8)\\x^{3} +9x^{2} +8x[/tex]

By factorization ,

[tex]x^{3} +9x^{2} +8x\\x(x^{3} +9x+8)[/tex]

Therefore, the greatest common factor of these terms are x

The greatest common factor (GCF) of a group of numbers has been the largest factor that each of the values shares in common.It refers to the largest number that could be divided evenly into two or smaller numbers.This component is a smaller number that divides evenly into the larger number.

Given Dimensions:

[tex]\to \bold{x, x+1,\ and\ x+8}[/tex]

Multiplying the three dimensions:

[tex]\to x(x+1)(x+8)\\\\\to x(x^2+ 8x+x+8)\\\\\to x(x^2+9x+8)\\\\\to x^3+9x^2+8x\\\\[/tex]

Therefore, the "greatest common factor" of the term is x.

Learn more:

Factor: brainly.com/question/18877132

which value of -7(x2-6)+2y when x=2 and y=6

Answers

Answer:

Solution

= -7 (x2 -6) +2y

= -7 ( 2*2 - 6 ) + 2 * 6

= -7 ( 4 - 6) + 12

= -7 - 2 + 12

= - 9 + 12

= 3

I hope this help u :)

Using the right triangle below, find the tangent of angle B.

A
600
0
3-13
300
B

Answers

Answer:1&3

Step-by-step explanation:

Solve for y
4x + 7y +5

Answers

To find the slope, m, of the line represented algebraically by the given linear equation in two variables: 4x = 7y + 5, first put the given equation in the slope-intercept form of the equation of a straight line, i.e., y = mx + b, as follows:

4x = 7y + 5 (Given)

4x - 5 = 7y + 5 - 5

4x - 5 = 7y + 0

4x - 5 = 7y

7y = 4x - 5 since equality is symmetric, i.e., if a = b, then b = a.

(7y)/7 = (4x - 5)/7

(7/7)y = (4x)/7 - 5/7

(1)y = (4/7)x- 5/7

y = (4/7)x - 5/7

From the last step above, the given equation is now in the slope-intercept form of the equation of a straight line: y = mx + b, i.e., y = (4/7)x - 5/7; therefore, we can now see by observation, that the slope of the line is m = 4/7 which means that the line has a positive slope, i.e., it's > 0, which means that if the line were to be constructed on a Cartesian coordinate system, it would slope upward from left to right and would intersect the y-axis at the point (0, -5/7).

2. The ratio of
the profit, cost of
materials and labour
in the production of
an article is 5:7:13
respectively. If the
cost of materials is Le
840 more than that of
labour, find the total
cost of producing the
article​

Answers

Answer:

3500

Step-by-step explanation:

Given the  ratio of  the profit, cost of  materials and labour  in the production of

an article to be 5:7:13 respectively, total ratio = 5+7+13 = 25

If the cost of labour is x, the cost of material will be 840+x (since the  cost of materials is Le  840 more than that of  labour)  .

Let the total cost of producing the article be y.

Cost of labour = 13/25 * y = x

Cost of labour  = 13y/25 = x.................... 1

Cost of material = 7/25*y = 840+x

Cost of material = 7y/25 = 840+x ..................... 2

From 1, 13y = 25x

x = 13y/25 ................... 3

Substituting equation 3 into 2:

7y/25 = 840+x

7y/25 = 840+13y/25

collect the like terms:

7y/25 - 13y/25 = 840

-6y/25 = 840

-6y = 25*840

y = 25*840/6

y = 3,500

Hence the total cost of producing the article is 3500

Given the following three points, find by hand the quadratic function they represent.
(-1,-8), (0, -1),(1,2)
(1 point)
Of(x) = -51% + 87 - 1
O f(x) = -3.2? + 4.1 - 1
Of(t) = -202 + 5x - 1
Of(1) = -3.1? + 10.1 - 1​

Answers

Answer:

The correct option is;

f(x) = -2·x² + 5·x - 1

Step-by-step explanation:

Given the points

(-1, -8), (0, -1), (1, 2), we have;

The general quadratic function;

f(x) = a·x² + b·x + c

From the given points, when x = -1, y = -8, which gives

-8 = a·(-1)² + b·(-1) + c = a - b + c

-8 =  a - b + c.....................................(1)

When x = 0, y = -1, which gives;

-1 = a·0² + b·0 + c = c

c = -1.....................................................(2)

When x = 1, y = 2, which gives;

2 = a·1² + b·1 + c = a + b + c...............(3)

Adding equation (1) to (3), gives;

-8 + 2 = a - b + c + a + b + c

-6 = 2·a + 2·c

From equation (2), c = -1, therefore;

-6 = 2·a + 2×(-1)

-2·a  = 2×(-1)+6 = -2 + 6 = 4

-2·a = 4

a = 4/-2 = -2

a = -2

From equation (1), we have;

-8 =  a - b + c = -2 - b - 1 = -3 - b

-8 + 3 = -b

-5 = -b

b = 5

The equation is therefore;

f(x) = -2·x² + 5·x - 1

The correct option is f(x) = -2·x² + 5·x - 1.

in angle PQR, angleQ=70 angle R=45 then angleP=.....

Answers

Answer:

65

Step-by-step explanation:

Because its a triangle,

P + Q + R = 180

So,

P + 70 + 45 = 180

P + 115 = 180

Subtract 115 on both sides

P = 65

factorize the expression:
mn² + mnp + 3mn + 3mp

Answers

Answer:

(mn+3m)(n+p)

Step-by-step explanation:

Answer:

(mn + 3m)(n + p)

Step-by-step explanation:

Factorize by grouping:

[tex]mn^2+mnp\\mn(n+p)\\\\3mn+3mp\\3m(n+p)\\\\(mn+3m)(n+p)[/tex]

The function f(x)=x^2 + ax + b has a minimum at (3,9) what are the values of a and b

Answers

Answer:

Step-by-step explanation:

The function is f(x)=x^2 + ax + b

Derivate the function:

● f'(x)= 2x + a

Solve the equation f'(x)=0 to find a

The minimum is at (3,9)

Replace x with 9

● 0 = 2×3 + a

● 0 = 6 + a

● a = -6

So the value of a is -6

Hence the equation is x^2 -6x+b

We have a khown point at (3,9)

● 9 = 3^2 -6×3 +b

● 9 = 9 -18 + b

● 9 = -9 +b

● b = 18

So the equation is x^2-6x+18

Verify by graphing the function.

The vetex is (3,9) and it is a minimum so the equation is right

Simplify 18 - 2[x + (x - 5)].
A) 13 - x
B) 13 - 4x
C) 8 - 4x
D) 28 - 4x

Answers

Answer:

Well the correct answer is −4x+28... But I don't

see that as one of your options. :/

Simplify:

18−2(x+x−5)

Distribute:

=18+(−2)(x)+(−2)(x)+(−2)(−5)

=18+−2x+−2x+10

Combine Like Terms:

=18+−2x+−2x+10

=(−2x+−2x)+(18+10)

=−4x+28

Answer:

−4x+28

Answer:

[tex] \boxed{ \bold{ \huge{ \boxed{ \sf{28 - 4x}}}}}[/tex]

Option D is the correct option.

Step-by-step explanation:

18 - 2 [ x + ( x - 5 ) ]

Remove the unnecessary Parentheses

⇒18 - 2 [ x + x - 5 ]

Collect like terms

⇒18 - 2 [ 2x - 5 ]

Distribute 2 through the parentheses

⇒18 - 4x + 10

Add the numbers

28 - 4x

Hope I helped!

Best regards!!

Find the value of x if 2x-1÷3 is greater than 1-3x÷4 by 1​

Answers

[tex]\\ \sf\longmapsto \dfrac{2x-1}{3}-\dfrac{1-3x}{4}=1[/tex]

[tex]\\ \sf\longmapsto \dfrac{4(2x-1)-3(1-3x)}{12}=1[/tex]

[tex]\\ \sf\longmapsto 8x-4-3+9x=12(1)[/tex]

[tex]\\ \sf\longmapsto 17x-7=12[/tex]

[tex]\\ \sf\longmapsto 17x=12+7[/tex]

[tex]\\ \sf\longmapsto 17x=19[/tex]

[tex]\\ \sf\longmapsto x=\dfrac{19}{17}[/tex]

2x-1/3+1=1-3x/4
5x-1/3=4
5x-1=12
5x=13
X=2.6

You pack sandwiches for a hike with your friends. Each sandwich takes 2 slices of bread, and each hiker eats one sandwich.

How many slices of bread are used for n hikers?

Answers

Given that:-

→ 1 sandwich = 2 slices of bread.

→ 1 hiker = 1 sandwich.

Then we have to find number of bread slices for n hikers .

→ Number of bread slices for 1 hiker = 2

→ Number of bread slices for 2 hikers = 2 × 2

→ For 3 hikers = 3 × 2

So in similar way

Number of bread slices for n hikers = 2×n 2n

So 2n is the answer.

Tori and Gavin were trying to solve the equation: (x+1)^2-3=13(x+1) 2 −3=13left parenthesis, x, plus, 1, right parenthesis, squared, minus, 3, equals, 13 Tori said, "I'll add 333 to both sides of the equation and solve using square roots." Gavin said, "I'll multiply (x+1)^2(x+1) 2 left parenthesis, x, plus, 1, right parenthesis, squared and rewrite the equation as x^2+2x+1-3=13x 2 +2x+1−3=13x, squared, plus, 2, x, plus, 1, minus, 3, equals, 13. Then I'll subtract 131313 from both sides, combine like terms, and solve using the quadratic formula with a=1a=1a, equals, 1, b=2b=2b, equals, 2, and c=-15c=−15c, equals, minus, 15."

Answers

The other answer is correct, its both !<3

Answer:

Both

Step-by-step explanation:

Both Tori and Gavin are correct, the two methods work. Completed this in Khan Academy, it's correct.

65000÷9330
400000÷9330
141200÷9330
800÷9330
Please give answers in smaller version and not how the calculator says​

Answers

Step-by-step explanation:

1.answer is 6.9667738478

2 answer is 42.872454448

3answer is 14.1339764202

4answer is 0.0857449089.

What is the domain of the relation (8, -2), (4,-2), (3, 2), (-5, -3)?
A. {8,4,3, -5}
B. {-8, -4, 3, 5)
C. 2-5, -3, 4, 8}
D. 2-3, -2, 2}

Answers

Answer:

A. {8, 4, 3, -5}

Step-by-step explanation:

The domain is the list of x values in a given function. Therefore, the domain is {8, 4, 3, -5}.

if your ans is correct i will choose you as a brainlist when the number of student of a school was increased by 30% it became 455. Find the previous number student.

Answers

Step-by-step explanation:

find 30% of 455

which is = 136.5

then subtract 136.5 from the original number(455)

455 - 136.5

=318.5 student

Previous number student was 350

Briefly explain how you would graph an equation such as y=7x-2

Answers

Answer:

put a mark at negative 2 on the graph then you would go up seven rigght one until you cant then you would go back to -2 and go down seven left one until you cant

Step-by-step explanation:

If 3sinA+4cosA=5 then find the value of cosA

Answers

Answer:

Cos A =1.4

Step-by-step explanation:

3sinA=5-4cosA; square both sides: 9sin^2(A)=25-40cosA+16cos^2(A);

9-9cos^2(A)=25-40cosA+16cos^2(A); 25cos^2(A)-40cosA+16=0=(5cosA-4)^2.

cosA=4/5 so sinA=3/5 (√(1-16/25) and sinA+cosA=7/5=1.4.

plz help me f(x) = 2x – 2 , find f(– 1)

Answers

Answer:

-4

Step-by-step explanation:

f(-1) = 2(-1) - 2 = -4

Plug in the f(-1) into f(x)=2x-2, which is f(-1)=2(-1)-2 and the answer is-4

11. The student government is selling flowers for homecoming. The project costs them $20 for advertising and
$3 for each flower sold.
a. Evaluate the expressions 3n+20 and 3(n+20) when n = 4.
b. Which expression shows their total cost? How do you know?

Answers

Answer:

32

Step-by-step explanation:

3(n+20) = 3(4+20) = 3(24) = 723n+20 = 3(4)+20= 12+20=32

If n represents the amount of flowers they sold, then the correct answer should be 32. The cost for each flower they sold would be 3 x 4 and then add $20 for advertising.

Other Questions
If the terminals of a battery with zero internal resistance are connected across two identical resistors in series, the total power delivered by the battery is 8.00 W. If the same battery is connected across the same resistors in parallel, what is the total power delivered by the battery Whats the largest fraction: 7/8, 5/8, 7/13, and 11/19 Intelligent beings in a distant galaxy send a signal to earth in the form of an electromagnetic wave. The frequency of the signal observed on earth is 2.2% greater than the frequency emitted by the source in the distant galaxy. What is the speed vrel of the galaxy relative to the earth (01.02 MC) Read and choose the option that answers the question. Hola, amigos! Me llamo Julieta y tengo quince aos. En las noches me bao a las ocho de la noche. Me lavo el pelo y me seco el pelo los domingos, martes y jueves. Despus, me pongo el pijama y me arreglo antes de salir del bao. Me duermo a las nueve y media de la noche. Based on the reading, select the "yo-go" verb used in the paragraph. (1 point) Wash Bathe Fix Have In a random sample of 20 NBA basketball games the mean number of points scored by the home team was 100.4 with a standard deviation of 4.86.Create and interpret a 95% confidence interval for the true mean number of points scored by an NBA basketball team at home.You and your friend were watching a LA Lakers game where they were not playing at home. They only scored 98 points. Your friend says, "Wow, I bet if they were playing at home they would have scored a lot more points." Do you agree or disagree with your friend? Support your detailed answer. Determine whether the following polygons are similar. How can I improve in non verbal communication The amount of money lana earns for tutoring is proportional to the time she spends tutoring. She earns $24 for tutoring 1 1/2 hours. what is the constant of proportionality for the relationship between dollars earned and number of hours? What is the area of the regular hexagon shown below? Hello! can anyone help me with this question which one is itneed help ASAP!!! A body makes a displacement of 4 m due East from a point O and then makes displacement of 3 m due North. Its resultant displacement from O Charles Cooley developed the idea that you learn about yourself from the feedback you receive from others who react to you. So if others tell you that you are funny, you believe you are funny. This construct is called: worth 10pts can someone help? Caribou Gold Mining Corporation is expected to pay a dividend of $6 in the upcoming year. Dividends are expected to decline at the rate of 3% per year. The risk-free rate of return is 5%, and the expected return on the market portfolio is 13%. The stock of Caribou Gold Mining Corporation has a beta of .5. Using the constant-growth DDM, the value of the stock is _________. A. $150 B. $50 C. $100 D. $200 1. What was the rhetorical situation of Stalin's radio broadcast? help me plzzzzz and ASAP. on a coordinate grid point P is at (4, 3) and point R is at (-2, -5) points Q and S are reflection of both points across the x-axis what are the coordinates of Q and S please answer correctly "A light beam incident on a diffraction grating consists of waves with two different wavelengths. The separation of the two first order lines is great if" Please help me to find this answer The reaction of butadiene gas (C4H6) with itself produces C8H12 gas as follows: The reaction is second order with a rate constant equal to 5.76 102 L/mol/min under certain conditions. If the initial concentration of butadiene is 0.200 M, what is the concentration in molarity remaining after 10.0 min? Report your answer to 3 decimal places.